LSAT and Law School Admissions Forum

Get expert LSAT preparation and law school admissions advice from PowerScore Test Preparation.

User avatar
 Dave Killoran
PowerScore Staff
  • PowerScore Staff
  • Posts: 5853
  • Joined: Mar 25, 2011
|
#88554
Complete Question Explanation
(The complete setup for this game can be found here: lsat/viewtopic.php?f=182&p=88548#p88548)

The correct answer choice is (C).

If K :longline: L, then Template #1 is in effect, with this modification:

G4-Q22-d1.png

However, because H :longline: L, from the third rule M :longline: F. This information changes the diagram above to:

G4-Q22-d2.png

Thus, M’s delivery is first, F’s and H’s are second and third (not necessarily in that order), G’s delivery is fourth, K’s delivery is fifth, and L’s delivery is sixth.

Consequently, answer choice (C) could be true and is correct.
You do not have the required permissions to view the files attached to this post.
 stsai
  • Posts: 27
  • Joined: Nov 05, 2011
|
#2587
These two templates are exactly the same as mine!
I reexamined the questions and realized that I got 19 and 20 wrong because I selected the "could be true" answers instead of what "must be true.
However, I am still unable to solve #22. Could you help me with this? Thanks!
User avatar
 Dave Killoran
PowerScore Staff
  • PowerScore Staff
  • Posts: 5853
  • Joined: Mar 25, 2011
|
#2600
Sure, let's break down #22 step-by-step:

1. The condition in the question stem establishes that K > L.


2. When K > L, the only template that can apply is Template #1.


3. When L is added to Template #1, the following initial diagram results:

Template #1:

F
- - - - - > G > K > L
M > H

Thus, we automatically know that G's delivery is fourth, K's is fifth, and L's is sixth.


4. Of course, the third rule is still in effect. From the contrapositive of that rule, because L is not earlier than H, then F cannot be earlier than M. Therefore, because only F or M is first, and F can't be first, we can conclude that M's delivery is first.


5. When the above information is combined, the only uncertainty is F and H (I'll present this order vertically due to the limitations of the text display):

1. M
2. F/H
3. H/F
4. G
5. K
6. L


6. Accordingly, answer choice (C) could be true and is thus correct.


Please let me know if that helps. Thanks!
 stsai
  • Posts: 27
  • Joined: Nov 05, 2011
|
#2626
Aha! I neglected Rule #3! That's why. Thanks so much, David!

Get the most out of your LSAT Prep Plus subscription.

Analyze and track your performance with our Testing and Analytics Package.